Showing that "zero vector space" is a vector space

Click For Summary
The discussion focuses on proving that the zero vector space, defined as ##\mathbb{V} = \{0\}##, is indeed a vector space. The user confirms that verifying the first six axioms is straightforward and seeks clarification on the distributive property, specifically whether the equation ##a(0 + 0) = a0 = 0 = a0 + a0## sufficiently demonstrates that scalar multiplication distributes over the zero vector. The response affirms that this approach is correct. Overall, the proof hinges on the unique properties of the zero vector in this space.
Mr Davis 97
Messages
1,461
Reaction score
44

Homework Statement


Let ## \mathbb{V} = \{0 \}## consist of a single vector ##0## and define ##0 + 0 = 0## and ##c0 = 0## for each scalar in ##\mathbb{F}##. Prove that ##\mathbb{V}## is a vector space.

Homework Equations

The Attempt at a Solution



Proving that the first six axioms of a vector space are true is trivial, I am just on the distributive axioms.

So if I want to show that ##a(x + y) = ax + ay## is true, where a is a scalar and x and y are vectors, is it sufficient to make the argument that ##a(0 + 0) = a0 = 0 = a0 + a0##? Does this show that a distributes over the vector ##0##?
 
Physics news on Phys.org
I guess ... since the space only contains one vector you can also show that the last two are equivalent to the previous ones.
ie. u=v, then a(u+v)=a(u+u)=2au = scalar multiplication axiom already tested.
ie u=v, then RHS: a(u+v)=2au and LHS=au+av = 2au so RHS=LHS
 
Mr Davis 97 said:
So if I want to show that ##a(x + y) = ax + ay## is true, where a is a scalar and x and y are vectors, is it sufficient to make the argument that ##a(0 + 0) = a0 = 0 = a0 + a0##? Does this show that a distributes over the vector ##0##?

Yes, that's exactly how to do it.
 
  • Like
Likes Mr Davis 97
Question: A clock's minute hand has length 4 and its hour hand has length 3. What is the distance between the tips at the moment when it is increasing most rapidly?(Putnam Exam Question) Answer: Making assumption that both the hands moves at constant angular velocities, the answer is ## \sqrt{7} .## But don't you think this assumption is somewhat doubtful and wrong?

Similar threads

  • · Replies 1 ·
Replies
1
Views
2K
Replies
9
Views
2K
Replies
7
Views
2K
  • · Replies 7 ·
Replies
7
Views
1K
  • · Replies 12 ·
Replies
12
Views
2K
Replies
2
Views
2K
  • · Replies 13 ·
Replies
13
Views
2K
  • · Replies 1 ·
Replies
1
Views
2K
  • · Replies 17 ·
Replies
17
Views
4K
  • · Replies 2 ·
Replies
2
Views
2K